LSAT and Law School Admissions Forum

Get expert LSAT preparation and law school admissions advice from PowerScore Test Preparation.

User avatar
 Dave Killoran
PowerScore Staff
  • PowerScore Staff
  • Posts: 5852
  • Joined: Mar 25, 2011
|
#26536
Complete Question Explanation

The correct answer choice is (D)


The condition in the question stem creates the following chain:
October 97_M12_L3_explanations_game#1_#2_diagram_1.png
Because there are four variables after H, H cannot fill any of the final four tracks, and thus H must be first, second, or third. But, because F is always second, we can determine that H is first or third, and that H is immediately before or immediately after F. Thus, answer choice (D) is correct.
You do not have the required permissions to view the files attached to this post.

Get the most out of your LSAT Prep Plus subscription.

Analyze and track your performance with our Testing and Analytics Package.